Merge remote branch 'public/master'
[course.git] / latex / problems / Serway_and_Jewett_4 / problem05.45.tex
1 \begin{problem*}{5.45}
2 A car rounds a banked curve as in Fig. 5.13.  The radius of curvature
3 of the road is $R$, the banking angle is $/theta$, and the coefficient
4 of static friction is $\mu_s$.
5 \Part{a} Determine the range of speeds the car can have without
6 slipping up or down the road.
7 \Part{b} Find the minimum value of $\mu_s$ such that the minimum
8 speed is zero.
9 \Part{c} What is the range of speeds possible if $R = 100\U{m}$,
10 $\theta = 10.0\dg$, and $\mu_s = 0.100$ (slippery conditions).
11 \end{problem*} % problem 5.45
12
13 \begin{solution}
14 Looking at Fig. 5.13 (text page 137) and adding friction, we see that
15 the forces on the car are friction $\vect{F}_f$, gravity $\vect{F}_g$,
16 and a normal force $\vect{F}_N$.  Let the vertical direction be
17 \jhat\ and the centerward direction to be \ihat, and the direction
18 centerward-down parallel to the surface of the road by \khat.  Let us
19 assume at first that $\vect{F}_N$ is in the $-\khat$ direction and at
20 its maximum possible value of $F_f = \mu_s F_N$.
21 \begin{align}
22  \sum F_\jhat &= F_N\cos\theta - mg + F_f\sin\theta= 0 \\
23  F_N (\cos\theta + \mu_s\sin\theta) &= mg \\
24  F_N &= \frac{mg}{\cos\theta + \mu_s\sin\theta} \\
25  \sum F_\ihat &= F_N\sin\theta - F_f\cos\theta
26            = F_N (\sin\theta - \mu_s\cos\theta) \\
27            &= \frac{mg}{\cos\theta + \mu_s\sin\theta} (\sin\theta - \mu_s\cos\theta) \\
28            &= mg \frac{\tan\theta - \mu_s}{1 + \mu_s\tan\theta}
29            = m\frac{v^2}{R} \label{eqn.45.Fi}\\
30  v &= \sqrt{ Rg\frac{\tan\theta - \mu_s}{1 + \mu_s\tan\theta} } \label{eqn.45.v}
31 \end{align}
32
33 \Part{a}
34 The work above shows that the minimum speed a car can have while going
35 around the turn is given by eqn \ref{eqn.45.v}, because that is the
36 case when friction is maximized in the $-\khat$ direction.  The
37 maximum speed that the car can have can be found by simply reversing
38 the sign of the frictional force above (so that $\vect{F}_f$ points in
39 the $+\khat$ direction), which we achieve by replacing any $\mu_s$s in
40 eqn \ref{eqn.45.v} with $(-\mu_s)$.  For any speeds between these
41 $F_f$ will be less than its maximum value of $\mu_s F_N$, and the car
42 will still not slip.  So
43 \begin{equation}
44  \ans{ \sqrt{ Rg\frac{\tan\theta - \mu_s}{1 + \mu_s\tan\theta} }
45        \le v \le
46        \sqrt{ Rg\frac{\tan\theta + \mu_s}{1 - \mu_s\tan\theta} } }
47 \end{equation}
48
49 \Part{b}
50 If the speed is 0, then $\vect{F}_f$ will be in the $-\khat$ direction
51 (opposing the $+\khat$ portion of $\vect{F}_g$).  Summing the forces
52 in the \khat direction we have
53 \begin{align}
54  \sum F_\khat &= F_g\sin\theta - F_N
55                = mg(\sin\theta - \mu_s\cos\theta)
56                = 0 \\
57  \mu_s &= \ans{\tan\theta}
58 \end{align}
59 Or we could go use eqn \ref{eqn.45.Fi}, our sum of forces in the
60 \ihat\ direction.
61 \begin{equation}
62  \sum F_\ihat = mg \frac{\tan\theta - \mu_s}{1 + \mu_s\tan\theta}
63               = m\frac{v^2}{R} = 0
64 \end{equation}
65 And set the numerator to $0$, which gives the same formula for $\mu_s$.
66
67 \Part{c}
68 Plugging into our ans for \Part{a} we have
69 \begin{align}
70  \sqrt{ 100\U{m} \cdot 9.8\U{m/s}^2 \frac{\tan 10.0\dg - 0.100}
71                                     {1 + 0.100 \cdot \tan 10.0\dg} }
72        &\le v \le
73  \sqrt{ 100\U{m} \cdot 9.8\U{m/s}^2 \frac{\tan 10.0\dg + 0.100}
74                                     {1 - 0.100 \cdot \tan 10.0\dg} } \\
75  \ans{ 8.57\U{m/s\ } } & \ans{ \le v \le 16.6\U{m/s} }
76 \end{align}
77 \end{solution}